LSAT and Law School Admissions Forum

Get expert LSAT preparation and law school admissions advice from PowerScore Test Preparation.

User avatar
 Dave Killoran
PowerScore Staff
  • PowerScore Staff
  • Posts: 5850
  • Joined: Mar 25, 2011
|
#85365
Complete Question Explanation
(The complete setup for this game can be found here: lsat/viewtopic.php?f=315&t=9170)

The correct answer choice is (C)

If H views the site on day 2, then from the final rule G must view the site on day 3, and from the first rule F must view the site on day 5:

G1-Q4-d1.png

From the contrapositive of the fourth rule, if L does not view the site on day 5, then K cannot view the site on day 4. This information eliminates answer choices (A), (D), and (E).

Only answer choices (B) and (C) remain in contention, and the difference between the two is J. As J can view the site on day 4, answer choice (C) is correct.
 srcline@noctrl.edu
  • Posts: 243
  • Joined: Oct 16, 2015
|
#23399
Hello

So this is where I initially ran into problems with this game. This question is stumping me.

So the inference that J day 1 :arrow: HG

So wouldn't the hypothetical read:

J H G,

How can Jones be in 4 then, if the other inference is K4 :arrow: L5 + F3? Wouldnt Knight have to be in spot 4?

THankyou
Sarah
 Nikki Siclunov
PowerScore Staff
  • PowerScore Staff
  • Posts: 1362
  • Joined: Aug 02, 2011
|
#23404
Sarah,

You are making a Mistaken Reversal of this inference. Just because the H2+G3 doesn't mean that J must be 1. J could be 1, but it doesn't have to. It can therefore be 4, along with M. In this particular question, K cannot be 4, because F is 5, not L. F is 5, in turn, because it cannot be 3 (G is).

Does that make sense?
 srcline@noctrl.edu
  • Posts: 243
  • Joined: Oct 16, 2015
|
#23406
Hello Nikki

Of course! I cant believe I did that, I had the dual option but for whatever reasons decided that J had to be in 1!

Thankyou for your quick and helpful reply.
Sarah
User avatar
 jailenea
  • Posts: 25
  • Joined: Aug 30, 2021
|
#90319
When doing this question, I ended up with 2 distributions. I narrowed it down to B and C. (Since C had bot Jones and Moss, I tested both in the diagram):
_ H G M F _ _
_ H G J F _ _

Nonetheless, I was unable to complete the rest of the diagram past this point by using the rules. There were too many unknowns. Thus, is C the answer because--up to this point--neither of these options (J/M) break any rules while sitting in position 4?
 Adam Tyson
PowerScore Staff
  • PowerScore Staff
  • Posts: 5153
  • Joined: Apr 14, 2011
|
#90320
Thanks for the question, jailenea! The issue here is the phrase "complete and accurate list." That means the correct answer will list every variable that could be 4th in this situation, and will not list any variable that cannot be 4th.

To prepare for the answers to this question you should build a local diagram, starting by placing H in the 2nd position per the question stem. Then, insert everything else that you know must be true in that circumstance. H at 2 forces G to 3 because of the HG block, and then F must be 5th because it can no longer be 3rd.

Now your local diagram should look like this:

_ H G _ F _ _

Next, it's time to prephrase the answer. What could be 4th with this setup? Not L, because it is never allowed to be 4th. Not K, because if K was 4th, L would be 5th, and L is not 5th because F is 5th. What does that leave us that could be 4th? Just J and M. Thus, the "complete and accurate list" of what could be 4th is J and M. You cannot leave one of them out, because then the list would not be complete! And you cannot include any other variable, because then the list would not be accurate!

Now that we know the complete and accurate list must be J and M, answer C is the winning answer. Answer B is incorrect because it is not complete without J. Answers A, D, and E are incorrect because including K makes them all inaccurate.

Get the most out of your LSAT Prep Plus subscription.

Analyze and track your performance with our Testing and Analytics Package.